Finding the area of inner triangle constructed by three cevian lines of a large triangleArea of triangle...

How can I create a Table like this in Latex?

When was drinking water recognized as crucial in marathon running?

Why doesn't "adolescent" take any articles in "listen to adolescent agonising"?

Wardrobe above a wall with fuse boxes

Book about a time-travel war fought by computers

What are all the squawk codes?

Finding the area of inner triangle constructed by three cevian lines of a large triangle

Why did John Williams use a march to symbolise Indiana Jones?

How to add an existing QGSVectorLayer to QGIS project?

Would the melodic leap of the opening phrase of Mozart's K545 be considered dissonant?

Is there any relevance to Thor getting his hair cut other than comedic value?

Why won't the strings command stop?

Plagiarism of code by other PhD student

How to substitute values from a list into a function?

Are small insurances worth it

Can a space-faring robot still function over a billion years?

How can neutral atoms have exactly zero electric field when there is a difference in the positions of the charges?

Is divide-by-zero a security vulnerability?

Is there a frame of reference in which I was born before I was conceived?

Where is the line between being obedient and getting bullied by a boss?

Difference between 'stomach' and 'uterus'

Misplaced tyre lever - alternatives?

1970s scifi/horror novel where protagonist is used by a crablike creature to feed its larvae, goes mad, and is defeated by retraumatising him

What are the issues with an additional (limited) concentration slot instead of Bladesong?



Finding the area of inner triangle constructed by three cevian lines of a large triangle


Area of triangle inside triangleCondition for three lines to be concurrent.How to find the area of the following isosceles triangleFind the area of the triangle AHCTriangle proofs and formulasCan I get the triangles area by squares of sides?Area of an inner triangle.Area of middle piece in triangleTrigonometric solution for finding the lengths of an obtuse triangle given the base, angle, & areaFinding the ratio of a side of $triangle ABC$ and its segment where one cevian line from the opposite vertex intersect the side in any point













3












$begingroup$


QUESTION:




In a triangle $ABC$, $AD, BE$ and $CF$ are three cevian lines such that $BD:DC = CE:AC = AF:FB = 3:1$. The area of $triangle ABC$ is $100$ unit$^2$. Find the area of $triangle HIG$






Attempt:



First, I thought that it can be done through applying Menelaus's Theorem. Despite having few knowledge about that theorem, I started like that:



As a consequence of Menelaus's theorem, if two cevian $BH$ and $GF$ of $ABG$ meet at $I$ then:



$$frac{HI}{IB} = frac{BF}{AB}.frac{AH}{HG}$$



$implies frac{HI}{IB} = frac{AH}{4HG}......(i)$



And



$$frac{FI}{IG} = frac{3HG}{AG}......(ii)$$



Similarly from $triangle ACI$, I got two more equations and that is:



$frac{CG}{GI} = frac{4GH}{AE}.......(iii)$



$frac{EH}{HI} = frac{IG}{4IC}........(iv)$



And likewise, from $triangle BHC$,



$frac{DG}{HG} = frac{3HI}{BH}........(v)$



$frac{IG}{GC} = frac{BI}{4IH}.........(vi)$



But later on, I thought that it wouldn't be so good for relating with so many sides or segments and also be ineffective instead of relating the area. Then, how could I relate the area using Menelaus's theorem or in any other way like pure geometry?



Thanks in advance.










share|cite|improve this question









$endgroup$








  • 2




    $begingroup$
    See Routh's Theorem.
    $endgroup$
    – Blue
    3 hours ago
















3












$begingroup$


QUESTION:




In a triangle $ABC$, $AD, BE$ and $CF$ are three cevian lines such that $BD:DC = CE:AC = AF:FB = 3:1$. The area of $triangle ABC$ is $100$ unit$^2$. Find the area of $triangle HIG$






Attempt:



First, I thought that it can be done through applying Menelaus's Theorem. Despite having few knowledge about that theorem, I started like that:



As a consequence of Menelaus's theorem, if two cevian $BH$ and $GF$ of $ABG$ meet at $I$ then:



$$frac{HI}{IB} = frac{BF}{AB}.frac{AH}{HG}$$



$implies frac{HI}{IB} = frac{AH}{4HG}......(i)$



And



$$frac{FI}{IG} = frac{3HG}{AG}......(ii)$$



Similarly from $triangle ACI$, I got two more equations and that is:



$frac{CG}{GI} = frac{4GH}{AE}.......(iii)$



$frac{EH}{HI} = frac{IG}{4IC}........(iv)$



And likewise, from $triangle BHC$,



$frac{DG}{HG} = frac{3HI}{BH}........(v)$



$frac{IG}{GC} = frac{BI}{4IH}.........(vi)$



But later on, I thought that it wouldn't be so good for relating with so many sides or segments and also be ineffective instead of relating the area. Then, how could I relate the area using Menelaus's theorem or in any other way like pure geometry?



Thanks in advance.










share|cite|improve this question









$endgroup$








  • 2




    $begingroup$
    See Routh's Theorem.
    $endgroup$
    – Blue
    3 hours ago














3












3








3


0



$begingroup$


QUESTION:




In a triangle $ABC$, $AD, BE$ and $CF$ are three cevian lines such that $BD:DC = CE:AC = AF:FB = 3:1$. The area of $triangle ABC$ is $100$ unit$^2$. Find the area of $triangle HIG$






Attempt:



First, I thought that it can be done through applying Menelaus's Theorem. Despite having few knowledge about that theorem, I started like that:



As a consequence of Menelaus's theorem, if two cevian $BH$ and $GF$ of $ABG$ meet at $I$ then:



$$frac{HI}{IB} = frac{BF}{AB}.frac{AH}{HG}$$



$implies frac{HI}{IB} = frac{AH}{4HG}......(i)$



And



$$frac{FI}{IG} = frac{3HG}{AG}......(ii)$$



Similarly from $triangle ACI$, I got two more equations and that is:



$frac{CG}{GI} = frac{4GH}{AE}.......(iii)$



$frac{EH}{HI} = frac{IG}{4IC}........(iv)$



And likewise, from $triangle BHC$,



$frac{DG}{HG} = frac{3HI}{BH}........(v)$



$frac{IG}{GC} = frac{BI}{4IH}.........(vi)$



But later on, I thought that it wouldn't be so good for relating with so many sides or segments and also be ineffective instead of relating the area. Then, how could I relate the area using Menelaus's theorem or in any other way like pure geometry?



Thanks in advance.










share|cite|improve this question









$endgroup$




QUESTION:




In a triangle $ABC$, $AD, BE$ and $CF$ are three cevian lines such that $BD:DC = CE:AC = AF:FB = 3:1$. The area of $triangle ABC$ is $100$ unit$^2$. Find the area of $triangle HIG$






Attempt:



First, I thought that it can be done through applying Menelaus's Theorem. Despite having few knowledge about that theorem, I started like that:



As a consequence of Menelaus's theorem, if two cevian $BH$ and $GF$ of $ABG$ meet at $I$ then:



$$frac{HI}{IB} = frac{BF}{AB}.frac{AH}{HG}$$



$implies frac{HI}{IB} = frac{AH}{4HG}......(i)$



And



$$frac{FI}{IG} = frac{3HG}{AG}......(ii)$$



Similarly from $triangle ACI$, I got two more equations and that is:



$frac{CG}{GI} = frac{4GH}{AE}.......(iii)$



$frac{EH}{HI} = frac{IG}{4IC}........(iv)$



And likewise, from $triangle BHC$,



$frac{DG}{HG} = frac{3HI}{BH}........(v)$



$frac{IG}{GC} = frac{BI}{4IH}.........(vi)$



But later on, I thought that it wouldn't be so good for relating with so many sides or segments and also be ineffective instead of relating the area. Then, how could I relate the area using Menelaus's theorem or in any other way like pure geometry?



Thanks in advance.







geometry contest-math triangle area plane-geometry






share|cite|improve this question













share|cite|improve this question











share|cite|improve this question




share|cite|improve this question










asked 3 hours ago









Anirban NiloyAnirban Niloy

7971218




7971218








  • 2




    $begingroup$
    See Routh's Theorem.
    $endgroup$
    – Blue
    3 hours ago














  • 2




    $begingroup$
    See Routh's Theorem.
    $endgroup$
    – Blue
    3 hours ago








2




2




$begingroup$
See Routh's Theorem.
$endgroup$
– Blue
3 hours ago




$begingroup$
See Routh's Theorem.
$endgroup$
– Blue
3 hours ago










3 Answers
3






active

oldest

votes


















2












$begingroup$

Let $Nin EC$ such that $DN||BE$ and $NC=y$.



Thus, by Thales $EN=3y$ and $AE=frac{1}{3}EC=frac{4}{3}y,$



which says
$$frac{AH}{HD}=frac{AE}{EN}=frac{frac{4}{3}y}{3y}=frac{4}{9}.$$
Also, let $Min DC$ such that $EM||AD$ and $DM=x$.



Thus, by Thales again we obtain: $MC=3x$, $BD=12x$ and
$$frac{BH}{HE}=frac{BD}{DM}=frac{12x}{x}=12.$$



Similarly, $$AG:GD=CI:IF=12:1$$ and
$$BI:IE=CG:GF=4:9,$$ which gives
$$AH:HG:GD=BI:IH:HE=CG:GI:IF=4:8:1.$$
Now, let $S_{Delta HIG}=s$.



Thus,
$$frac{S_{Delta GFA}}{s}=frac{9cdot12}{8cdot8}=frac{27}{16},$$
which gives
$$S_{Delta GFA}=frac{27}{16}s.$$
Also,
$$frac{S_{Delta AFC}}{frac{27}{16}s}=frac{FC}{FG}=frac{13}{9},$$ which gives
$$S_{Delta AFC}=frac{39}{16}s$$ and since
$$frac{S_{Delta ABC}}{frac{39}{16}s}=frac{4}{3},$$ we obtain:
$$S_{Delta ABC}=frac{13}{4}s$$ and $$s=frac{400}{13}.$$






share|cite|improve this answer









$endgroup$





















    2












    $begingroup$

    (Adapted from my proof of the side-trisecting version on AoPS)



    First, invert the logic of the construction. Start with an arbitrary triangle $GHI$, and extend $GH$ to $A$ so that $frac{HA}{GH}=frac12$, $HI$ to $B$ so that $frac{IB}{HI}=frac12$, and $IG$ to $C$ so that $frac{GC}{IG}=frac12$. Then, extend $AG$ to meet $BC$ at $D$, $BH$ to meet $AC$ at $E$, and $HI$ to meet $AB$ at $F$. We wish to find the ratio of the area of triangle $GHI$ to that of $ABC$.



    Figure 1



    (Unlike the figure in the question or that in Anirban Niloy's answer, this figure is actually to scale)



    From $GH=2cdot HA$ and the common vertex $I$, the red triangle $GHI$ has twice the area of the yellow triangle $HAI$. Similarly, $GHI$ has twice the area of the other two yellow triangles $IBG$ and $GCH$.

    From $GH=2cdot HA$ and the common vertex $C$, the yellow triangle $GHC$ has twice the area of the green triangle $HAC$. Similarly, $HIA$ has twice the area of $IBA$ and $IGB$ has twice the area of $GCB$, so the area of each yellow triangle is twice that of each green triangle.



    With three green triangles, three yellow triangles, and one red triangle, the total area is $3+3cdot 2+1cdot 4=13$ times the area of one green triangle. The area of the red triangle $GHI$ is $4$ times the area of a green triangle, for a ratio of $frac{4}{13}$ of the large triangle $ABC$.



    That's the area ratios. But we're not done; we still need to show that this is the same as the original configuration. The ratio $frac{AE}{EC}$ is equal to the ratio of areas $frac{ABE}{CBE}$, which is equal to the ratio of areas $frac{ABH}{CBH}$ - one green and one yellow triangle over one green, two yellow, and one red triangle. That's $1+2=3$ times a green triangle in the numerator, and $1+2cdot 2+4=9$ times a green triangle in the denominator, a ratio of $frac13$. Similarly, $frac{BF}{FA}=frac13$ and $frac{CD}{DB}=frac13$. The sides are indeed cut in fourths, and it's the same configuration.



    Applying this to the given total area of $100$, the central triangle's area is $frac{4}{13}cdot 100=frac{400}{13}$.






    share|cite|improve this answer











    $endgroup$





















      1












      $begingroup$

      I found a solution but actually it's not mine. I found it on a book and noticed that it is quite different from the Routh's theorem and it has been solved by the pure geometic way. And that is:





      Point $C$ and $H$ are connected.



      $frac{triangle ADC}{triangle ADB} = frac{1}{3}$ and $frac{triangle HDC}{triangle HDB} = frac{1}{3}$



      Hence, $$frac{triangle ACH}{triangle ABH} = frac{1}{3}$$.



      Similarly, $$frac{triangle CHB}{triangle ABH} = 3$$



      Now, $triangle ABC = triangle ABH + triangle ACH +triangle BHC = triangle ABH + frac{1}{3}triangle ABH + triangle ABH = frac{13}{3} triangle ABH$



      So, $$triangle ABH = frac{3}{13} triangle ABC$$



      Similarly, $$triangle ACG = frac{3}{13} triangle ABC$$



      And, $$triangle BIC = frac{3}{13} triangle ABC$$



      Therfore, $triangle HIG = triangle ABC - triangle ACG - triangle BIC - triangle ABH = (1-3*frac{3}{13})triangle ABC = frac{4}{13}*100 = frac{400}{13}$



      Hence, we get the area of $triangle HIG = frac{400}{13}$ unit$^2$.






      share|cite|improve this answer









      $endgroup$













        Your Answer





        StackExchange.ifUsing("editor", function () {
        return StackExchange.using("mathjaxEditing", function () {
        StackExchange.MarkdownEditor.creationCallbacks.add(function (editor, postfix) {
        StackExchange.mathjaxEditing.prepareWmdForMathJax(editor, postfix, [["$", "$"], ["\\(","\\)"]]);
        });
        });
        }, "mathjax-editing");

        StackExchange.ready(function() {
        var channelOptions = {
        tags: "".split(" "),
        id: "69"
        };
        initTagRenderer("".split(" "), "".split(" "), channelOptions);

        StackExchange.using("externalEditor", function() {
        // Have to fire editor after snippets, if snippets enabled
        if (StackExchange.settings.snippets.snippetsEnabled) {
        StackExchange.using("snippets", function() {
        createEditor();
        });
        }
        else {
        createEditor();
        }
        });

        function createEditor() {
        StackExchange.prepareEditor({
        heartbeatType: 'answer',
        autoActivateHeartbeat: false,
        convertImagesToLinks: true,
        noModals: true,
        showLowRepImageUploadWarning: true,
        reputationToPostImages: 10,
        bindNavPrevention: true,
        postfix: "",
        imageUploader: {
        brandingHtml: "Powered by u003ca class="icon-imgur-white" href="https://imgur.com/"u003eu003c/au003e",
        contentPolicyHtml: "User contributions licensed under u003ca href="https://creativecommons.org/licenses/by-sa/3.0/"u003ecc by-sa 3.0 with attribution requiredu003c/au003e u003ca href="https://stackoverflow.com/legal/content-policy"u003e(content policy)u003c/au003e",
        allowUrls: true
        },
        noCode: true, onDemand: true,
        discardSelector: ".discard-answer"
        ,immediatelyShowMarkdownHelp:true
        });


        }
        });














        draft saved

        draft discarded


















        StackExchange.ready(
        function () {
        StackExchange.openid.initPostLogin('.new-post-login', 'https%3a%2f%2fmath.stackexchange.com%2fquestions%2f3137440%2ffinding-the-area-of-inner-triangle-constructed-by-three-cevian-lines-of-a-large%23new-answer', 'question_page');
        }
        );

        Post as a guest















        Required, but never shown

























        3 Answers
        3






        active

        oldest

        votes








        3 Answers
        3






        active

        oldest

        votes









        active

        oldest

        votes






        active

        oldest

        votes









        2












        $begingroup$

        Let $Nin EC$ such that $DN||BE$ and $NC=y$.



        Thus, by Thales $EN=3y$ and $AE=frac{1}{3}EC=frac{4}{3}y,$



        which says
        $$frac{AH}{HD}=frac{AE}{EN}=frac{frac{4}{3}y}{3y}=frac{4}{9}.$$
        Also, let $Min DC$ such that $EM||AD$ and $DM=x$.



        Thus, by Thales again we obtain: $MC=3x$, $BD=12x$ and
        $$frac{BH}{HE}=frac{BD}{DM}=frac{12x}{x}=12.$$



        Similarly, $$AG:GD=CI:IF=12:1$$ and
        $$BI:IE=CG:GF=4:9,$$ which gives
        $$AH:HG:GD=BI:IH:HE=CG:GI:IF=4:8:1.$$
        Now, let $S_{Delta HIG}=s$.



        Thus,
        $$frac{S_{Delta GFA}}{s}=frac{9cdot12}{8cdot8}=frac{27}{16},$$
        which gives
        $$S_{Delta GFA}=frac{27}{16}s.$$
        Also,
        $$frac{S_{Delta AFC}}{frac{27}{16}s}=frac{FC}{FG}=frac{13}{9},$$ which gives
        $$S_{Delta AFC}=frac{39}{16}s$$ and since
        $$frac{S_{Delta ABC}}{frac{39}{16}s}=frac{4}{3},$$ we obtain:
        $$S_{Delta ABC}=frac{13}{4}s$$ and $$s=frac{400}{13}.$$






        share|cite|improve this answer









        $endgroup$


















          2












          $begingroup$

          Let $Nin EC$ such that $DN||BE$ and $NC=y$.



          Thus, by Thales $EN=3y$ and $AE=frac{1}{3}EC=frac{4}{3}y,$



          which says
          $$frac{AH}{HD}=frac{AE}{EN}=frac{frac{4}{3}y}{3y}=frac{4}{9}.$$
          Also, let $Min DC$ such that $EM||AD$ and $DM=x$.



          Thus, by Thales again we obtain: $MC=3x$, $BD=12x$ and
          $$frac{BH}{HE}=frac{BD}{DM}=frac{12x}{x}=12.$$



          Similarly, $$AG:GD=CI:IF=12:1$$ and
          $$BI:IE=CG:GF=4:9,$$ which gives
          $$AH:HG:GD=BI:IH:HE=CG:GI:IF=4:8:1.$$
          Now, let $S_{Delta HIG}=s$.



          Thus,
          $$frac{S_{Delta GFA}}{s}=frac{9cdot12}{8cdot8}=frac{27}{16},$$
          which gives
          $$S_{Delta GFA}=frac{27}{16}s.$$
          Also,
          $$frac{S_{Delta AFC}}{frac{27}{16}s}=frac{FC}{FG}=frac{13}{9},$$ which gives
          $$S_{Delta AFC}=frac{39}{16}s$$ and since
          $$frac{S_{Delta ABC}}{frac{39}{16}s}=frac{4}{3},$$ we obtain:
          $$S_{Delta ABC}=frac{13}{4}s$$ and $$s=frac{400}{13}.$$






          share|cite|improve this answer









          $endgroup$
















            2












            2








            2





            $begingroup$

            Let $Nin EC$ such that $DN||BE$ and $NC=y$.



            Thus, by Thales $EN=3y$ and $AE=frac{1}{3}EC=frac{4}{3}y,$



            which says
            $$frac{AH}{HD}=frac{AE}{EN}=frac{frac{4}{3}y}{3y}=frac{4}{9}.$$
            Also, let $Min DC$ such that $EM||AD$ and $DM=x$.



            Thus, by Thales again we obtain: $MC=3x$, $BD=12x$ and
            $$frac{BH}{HE}=frac{BD}{DM}=frac{12x}{x}=12.$$



            Similarly, $$AG:GD=CI:IF=12:1$$ and
            $$BI:IE=CG:GF=4:9,$$ which gives
            $$AH:HG:GD=BI:IH:HE=CG:GI:IF=4:8:1.$$
            Now, let $S_{Delta HIG}=s$.



            Thus,
            $$frac{S_{Delta GFA}}{s}=frac{9cdot12}{8cdot8}=frac{27}{16},$$
            which gives
            $$S_{Delta GFA}=frac{27}{16}s.$$
            Also,
            $$frac{S_{Delta AFC}}{frac{27}{16}s}=frac{FC}{FG}=frac{13}{9},$$ which gives
            $$S_{Delta AFC}=frac{39}{16}s$$ and since
            $$frac{S_{Delta ABC}}{frac{39}{16}s}=frac{4}{3},$$ we obtain:
            $$S_{Delta ABC}=frac{13}{4}s$$ and $$s=frac{400}{13}.$$






            share|cite|improve this answer









            $endgroup$



            Let $Nin EC$ such that $DN||BE$ and $NC=y$.



            Thus, by Thales $EN=3y$ and $AE=frac{1}{3}EC=frac{4}{3}y,$



            which says
            $$frac{AH}{HD}=frac{AE}{EN}=frac{frac{4}{3}y}{3y}=frac{4}{9}.$$
            Also, let $Min DC$ such that $EM||AD$ and $DM=x$.



            Thus, by Thales again we obtain: $MC=3x$, $BD=12x$ and
            $$frac{BH}{HE}=frac{BD}{DM}=frac{12x}{x}=12.$$



            Similarly, $$AG:GD=CI:IF=12:1$$ and
            $$BI:IE=CG:GF=4:9,$$ which gives
            $$AH:HG:GD=BI:IH:HE=CG:GI:IF=4:8:1.$$
            Now, let $S_{Delta HIG}=s$.



            Thus,
            $$frac{S_{Delta GFA}}{s}=frac{9cdot12}{8cdot8}=frac{27}{16},$$
            which gives
            $$S_{Delta GFA}=frac{27}{16}s.$$
            Also,
            $$frac{S_{Delta AFC}}{frac{27}{16}s}=frac{FC}{FG}=frac{13}{9},$$ which gives
            $$S_{Delta AFC}=frac{39}{16}s$$ and since
            $$frac{S_{Delta ABC}}{frac{39}{16}s}=frac{4}{3},$$ we obtain:
            $$S_{Delta ABC}=frac{13}{4}s$$ and $$s=frac{400}{13}.$$







            share|cite|improve this answer












            share|cite|improve this answer



            share|cite|improve this answer










            answered 2 hours ago









            Michael RozenbergMichael Rozenberg

            107k1894198




            107k1894198























                2












                $begingroup$

                (Adapted from my proof of the side-trisecting version on AoPS)



                First, invert the logic of the construction. Start with an arbitrary triangle $GHI$, and extend $GH$ to $A$ so that $frac{HA}{GH}=frac12$, $HI$ to $B$ so that $frac{IB}{HI}=frac12$, and $IG$ to $C$ so that $frac{GC}{IG}=frac12$. Then, extend $AG$ to meet $BC$ at $D$, $BH$ to meet $AC$ at $E$, and $HI$ to meet $AB$ at $F$. We wish to find the ratio of the area of triangle $GHI$ to that of $ABC$.



                Figure 1



                (Unlike the figure in the question or that in Anirban Niloy's answer, this figure is actually to scale)



                From $GH=2cdot HA$ and the common vertex $I$, the red triangle $GHI$ has twice the area of the yellow triangle $HAI$. Similarly, $GHI$ has twice the area of the other two yellow triangles $IBG$ and $GCH$.

                From $GH=2cdot HA$ and the common vertex $C$, the yellow triangle $GHC$ has twice the area of the green triangle $HAC$. Similarly, $HIA$ has twice the area of $IBA$ and $IGB$ has twice the area of $GCB$, so the area of each yellow triangle is twice that of each green triangle.



                With three green triangles, three yellow triangles, and one red triangle, the total area is $3+3cdot 2+1cdot 4=13$ times the area of one green triangle. The area of the red triangle $GHI$ is $4$ times the area of a green triangle, for a ratio of $frac{4}{13}$ of the large triangle $ABC$.



                That's the area ratios. But we're not done; we still need to show that this is the same as the original configuration. The ratio $frac{AE}{EC}$ is equal to the ratio of areas $frac{ABE}{CBE}$, which is equal to the ratio of areas $frac{ABH}{CBH}$ - one green and one yellow triangle over one green, two yellow, and one red triangle. That's $1+2=3$ times a green triangle in the numerator, and $1+2cdot 2+4=9$ times a green triangle in the denominator, a ratio of $frac13$. Similarly, $frac{BF}{FA}=frac13$ and $frac{CD}{DB}=frac13$. The sides are indeed cut in fourths, and it's the same configuration.



                Applying this to the given total area of $100$, the central triangle's area is $frac{4}{13}cdot 100=frac{400}{13}$.






                share|cite|improve this answer











                $endgroup$


















                  2












                  $begingroup$

                  (Adapted from my proof of the side-trisecting version on AoPS)



                  First, invert the logic of the construction. Start with an arbitrary triangle $GHI$, and extend $GH$ to $A$ so that $frac{HA}{GH}=frac12$, $HI$ to $B$ so that $frac{IB}{HI}=frac12$, and $IG$ to $C$ so that $frac{GC}{IG}=frac12$. Then, extend $AG$ to meet $BC$ at $D$, $BH$ to meet $AC$ at $E$, and $HI$ to meet $AB$ at $F$. We wish to find the ratio of the area of triangle $GHI$ to that of $ABC$.



                  Figure 1



                  (Unlike the figure in the question or that in Anirban Niloy's answer, this figure is actually to scale)



                  From $GH=2cdot HA$ and the common vertex $I$, the red triangle $GHI$ has twice the area of the yellow triangle $HAI$. Similarly, $GHI$ has twice the area of the other two yellow triangles $IBG$ and $GCH$.

                  From $GH=2cdot HA$ and the common vertex $C$, the yellow triangle $GHC$ has twice the area of the green triangle $HAC$. Similarly, $HIA$ has twice the area of $IBA$ and $IGB$ has twice the area of $GCB$, so the area of each yellow triangle is twice that of each green triangle.



                  With three green triangles, three yellow triangles, and one red triangle, the total area is $3+3cdot 2+1cdot 4=13$ times the area of one green triangle. The area of the red triangle $GHI$ is $4$ times the area of a green triangle, for a ratio of $frac{4}{13}$ of the large triangle $ABC$.



                  That's the area ratios. But we're not done; we still need to show that this is the same as the original configuration. The ratio $frac{AE}{EC}$ is equal to the ratio of areas $frac{ABE}{CBE}$, which is equal to the ratio of areas $frac{ABH}{CBH}$ - one green and one yellow triangle over one green, two yellow, and one red triangle. That's $1+2=3$ times a green triangle in the numerator, and $1+2cdot 2+4=9$ times a green triangle in the denominator, a ratio of $frac13$. Similarly, $frac{BF}{FA}=frac13$ and $frac{CD}{DB}=frac13$. The sides are indeed cut in fourths, and it's the same configuration.



                  Applying this to the given total area of $100$, the central triangle's area is $frac{4}{13}cdot 100=frac{400}{13}$.






                  share|cite|improve this answer











                  $endgroup$
















                    2












                    2








                    2





                    $begingroup$

                    (Adapted from my proof of the side-trisecting version on AoPS)



                    First, invert the logic of the construction. Start with an arbitrary triangle $GHI$, and extend $GH$ to $A$ so that $frac{HA}{GH}=frac12$, $HI$ to $B$ so that $frac{IB}{HI}=frac12$, and $IG$ to $C$ so that $frac{GC}{IG}=frac12$. Then, extend $AG$ to meet $BC$ at $D$, $BH$ to meet $AC$ at $E$, and $HI$ to meet $AB$ at $F$. We wish to find the ratio of the area of triangle $GHI$ to that of $ABC$.



                    Figure 1



                    (Unlike the figure in the question or that in Anirban Niloy's answer, this figure is actually to scale)



                    From $GH=2cdot HA$ and the common vertex $I$, the red triangle $GHI$ has twice the area of the yellow triangle $HAI$. Similarly, $GHI$ has twice the area of the other two yellow triangles $IBG$ and $GCH$.

                    From $GH=2cdot HA$ and the common vertex $C$, the yellow triangle $GHC$ has twice the area of the green triangle $HAC$. Similarly, $HIA$ has twice the area of $IBA$ and $IGB$ has twice the area of $GCB$, so the area of each yellow triangle is twice that of each green triangle.



                    With three green triangles, three yellow triangles, and one red triangle, the total area is $3+3cdot 2+1cdot 4=13$ times the area of one green triangle. The area of the red triangle $GHI$ is $4$ times the area of a green triangle, for a ratio of $frac{4}{13}$ of the large triangle $ABC$.



                    That's the area ratios. But we're not done; we still need to show that this is the same as the original configuration. The ratio $frac{AE}{EC}$ is equal to the ratio of areas $frac{ABE}{CBE}$, which is equal to the ratio of areas $frac{ABH}{CBH}$ - one green and one yellow triangle over one green, two yellow, and one red triangle. That's $1+2=3$ times a green triangle in the numerator, and $1+2cdot 2+4=9$ times a green triangle in the denominator, a ratio of $frac13$. Similarly, $frac{BF}{FA}=frac13$ and $frac{CD}{DB}=frac13$. The sides are indeed cut in fourths, and it's the same configuration.



                    Applying this to the given total area of $100$, the central triangle's area is $frac{4}{13}cdot 100=frac{400}{13}$.






                    share|cite|improve this answer











                    $endgroup$



                    (Adapted from my proof of the side-trisecting version on AoPS)



                    First, invert the logic of the construction. Start with an arbitrary triangle $GHI$, and extend $GH$ to $A$ so that $frac{HA}{GH}=frac12$, $HI$ to $B$ so that $frac{IB}{HI}=frac12$, and $IG$ to $C$ so that $frac{GC}{IG}=frac12$. Then, extend $AG$ to meet $BC$ at $D$, $BH$ to meet $AC$ at $E$, and $HI$ to meet $AB$ at $F$. We wish to find the ratio of the area of triangle $GHI$ to that of $ABC$.



                    Figure 1



                    (Unlike the figure in the question or that in Anirban Niloy's answer, this figure is actually to scale)



                    From $GH=2cdot HA$ and the common vertex $I$, the red triangle $GHI$ has twice the area of the yellow triangle $HAI$. Similarly, $GHI$ has twice the area of the other two yellow triangles $IBG$ and $GCH$.

                    From $GH=2cdot HA$ and the common vertex $C$, the yellow triangle $GHC$ has twice the area of the green triangle $HAC$. Similarly, $HIA$ has twice the area of $IBA$ and $IGB$ has twice the area of $GCB$, so the area of each yellow triangle is twice that of each green triangle.



                    With three green triangles, three yellow triangles, and one red triangle, the total area is $3+3cdot 2+1cdot 4=13$ times the area of one green triangle. The area of the red triangle $GHI$ is $4$ times the area of a green triangle, for a ratio of $frac{4}{13}$ of the large triangle $ABC$.



                    That's the area ratios. But we're not done; we still need to show that this is the same as the original configuration. The ratio $frac{AE}{EC}$ is equal to the ratio of areas $frac{ABE}{CBE}$, which is equal to the ratio of areas $frac{ABH}{CBH}$ - one green and one yellow triangle over one green, two yellow, and one red triangle. That's $1+2=3$ times a green triangle in the numerator, and $1+2cdot 2+4=9$ times a green triangle in the denominator, a ratio of $frac13$. Similarly, $frac{BF}{FA}=frac13$ and $frac{CD}{DB}=frac13$. The sides are indeed cut in fourths, and it's the same configuration.



                    Applying this to the given total area of $100$, the central triangle's area is $frac{4}{13}cdot 100=frac{400}{13}$.







                    share|cite|improve this answer














                    share|cite|improve this answer



                    share|cite|improve this answer








                    edited 48 mins ago

























                    answered 1 hour ago









                    jmerryjmerry

                    12.2k1628




                    12.2k1628























                        1












                        $begingroup$

                        I found a solution but actually it's not mine. I found it on a book and noticed that it is quite different from the Routh's theorem and it has been solved by the pure geometic way. And that is:





                        Point $C$ and $H$ are connected.



                        $frac{triangle ADC}{triangle ADB} = frac{1}{3}$ and $frac{triangle HDC}{triangle HDB} = frac{1}{3}$



                        Hence, $$frac{triangle ACH}{triangle ABH} = frac{1}{3}$$.



                        Similarly, $$frac{triangle CHB}{triangle ABH} = 3$$



                        Now, $triangle ABC = triangle ABH + triangle ACH +triangle BHC = triangle ABH + frac{1}{3}triangle ABH + triangle ABH = frac{13}{3} triangle ABH$



                        So, $$triangle ABH = frac{3}{13} triangle ABC$$



                        Similarly, $$triangle ACG = frac{3}{13} triangle ABC$$



                        And, $$triangle BIC = frac{3}{13} triangle ABC$$



                        Therfore, $triangle HIG = triangle ABC - triangle ACG - triangle BIC - triangle ABH = (1-3*frac{3}{13})triangle ABC = frac{4}{13}*100 = frac{400}{13}$



                        Hence, we get the area of $triangle HIG = frac{400}{13}$ unit$^2$.






                        share|cite|improve this answer









                        $endgroup$


















                          1












                          $begingroup$

                          I found a solution but actually it's not mine. I found it on a book and noticed that it is quite different from the Routh's theorem and it has been solved by the pure geometic way. And that is:





                          Point $C$ and $H$ are connected.



                          $frac{triangle ADC}{triangle ADB} = frac{1}{3}$ and $frac{triangle HDC}{triangle HDB} = frac{1}{3}$



                          Hence, $$frac{triangle ACH}{triangle ABH} = frac{1}{3}$$.



                          Similarly, $$frac{triangle CHB}{triangle ABH} = 3$$



                          Now, $triangle ABC = triangle ABH + triangle ACH +triangle BHC = triangle ABH + frac{1}{3}triangle ABH + triangle ABH = frac{13}{3} triangle ABH$



                          So, $$triangle ABH = frac{3}{13} triangle ABC$$



                          Similarly, $$triangle ACG = frac{3}{13} triangle ABC$$



                          And, $$triangle BIC = frac{3}{13} triangle ABC$$



                          Therfore, $triangle HIG = triangle ABC - triangle ACG - triangle BIC - triangle ABH = (1-3*frac{3}{13})triangle ABC = frac{4}{13}*100 = frac{400}{13}$



                          Hence, we get the area of $triangle HIG = frac{400}{13}$ unit$^2$.






                          share|cite|improve this answer









                          $endgroup$
















                            1












                            1








                            1





                            $begingroup$

                            I found a solution but actually it's not mine. I found it on a book and noticed that it is quite different from the Routh's theorem and it has been solved by the pure geometic way. And that is:





                            Point $C$ and $H$ are connected.



                            $frac{triangle ADC}{triangle ADB} = frac{1}{3}$ and $frac{triangle HDC}{triangle HDB} = frac{1}{3}$



                            Hence, $$frac{triangle ACH}{triangle ABH} = frac{1}{3}$$.



                            Similarly, $$frac{triangle CHB}{triangle ABH} = 3$$



                            Now, $triangle ABC = triangle ABH + triangle ACH +triangle BHC = triangle ABH + frac{1}{3}triangle ABH + triangle ABH = frac{13}{3} triangle ABH$



                            So, $$triangle ABH = frac{3}{13} triangle ABC$$



                            Similarly, $$triangle ACG = frac{3}{13} triangle ABC$$



                            And, $$triangle BIC = frac{3}{13} triangle ABC$$



                            Therfore, $triangle HIG = triangle ABC - triangle ACG - triangle BIC - triangle ABH = (1-3*frac{3}{13})triangle ABC = frac{4}{13}*100 = frac{400}{13}$



                            Hence, we get the area of $triangle HIG = frac{400}{13}$ unit$^2$.






                            share|cite|improve this answer









                            $endgroup$



                            I found a solution but actually it's not mine. I found it on a book and noticed that it is quite different from the Routh's theorem and it has been solved by the pure geometic way. And that is:





                            Point $C$ and $H$ are connected.



                            $frac{triangle ADC}{triangle ADB} = frac{1}{3}$ and $frac{triangle HDC}{triangle HDB} = frac{1}{3}$



                            Hence, $$frac{triangle ACH}{triangle ABH} = frac{1}{3}$$.



                            Similarly, $$frac{triangle CHB}{triangle ABH} = 3$$



                            Now, $triangle ABC = triangle ABH + triangle ACH +triangle BHC = triangle ABH + frac{1}{3}triangle ABH + triangle ABH = frac{13}{3} triangle ABH$



                            So, $$triangle ABH = frac{3}{13} triangle ABC$$



                            Similarly, $$triangle ACG = frac{3}{13} triangle ABC$$



                            And, $$triangle BIC = frac{3}{13} triangle ABC$$



                            Therfore, $triangle HIG = triangle ABC - triangle ACG - triangle BIC - triangle ABH = (1-3*frac{3}{13})triangle ABC = frac{4}{13}*100 = frac{400}{13}$



                            Hence, we get the area of $triangle HIG = frac{400}{13}$ unit$^2$.







                            share|cite|improve this answer












                            share|cite|improve this answer



                            share|cite|improve this answer










                            answered 2 hours ago









                            Anirban NiloyAnirban Niloy

                            7971218




                            7971218






























                                draft saved

                                draft discarded




















































                                Thanks for contributing an answer to Mathematics Stack Exchange!


                                • Please be sure to answer the question. Provide details and share your research!

                                But avoid



                                • Asking for help, clarification, or responding to other answers.

                                • Making statements based on opinion; back them up with references or personal experience.


                                Use MathJax to format equations. MathJax reference.


                                To learn more, see our tips on writing great answers.




                                draft saved


                                draft discarded














                                StackExchange.ready(
                                function () {
                                StackExchange.openid.initPostLogin('.new-post-login', 'https%3a%2f%2fmath.stackexchange.com%2fquestions%2f3137440%2ffinding-the-area-of-inner-triangle-constructed-by-three-cevian-lines-of-a-large%23new-answer', 'question_page');
                                }
                                );

                                Post as a guest















                                Required, but never shown





















































                                Required, but never shown














                                Required, but never shown












                                Required, but never shown







                                Required, but never shown

































                                Required, but never shown














                                Required, but never shown












                                Required, but never shown







                                Required, but never shown







                                Popular posts from this blog

                                As a Security Precaution, the user account has been locked The Next CEO of Stack OverflowMS...

                                Список ссавців Італії Природоохоронні статуси | Список |...

                                Українські прізвища Зміст Історичні відомості |...